A primera vista, uno puede decir "¿por qué no utilizar un solo término, o tal vez tres o más términos? ¿Por qué utilizar dos términos? Veo que los libros (ver por ejemplo buenos libros como la mecánica cuántica de Griffiths o la química física de Atkins) utilizan el $\ln(x!)\approx x \ln x - x$ fórmula, pero nadie justifica esta elección. Hice el cálculo utilizando la fórmula más pequeña $\ln(x!)\approx x \ln x$ encontrando, para fermiones y bosones idénticos, los mismos resultados que los libros encuentran utilizando la fórmula más larga (es decir $N_n = \frac{d_n}{e^{\alpha+\beta e_n} \pm 1}$ donde $N_n$ es el número de partículas en el nivel de energía $E_n$ con degeneración $d_n$ , mientras que $\alpha$ y $\beta$ son los multiplicadores de Lagrange).
La única razón que puedo especular para elegir $\ln(x!)\approx x \ln x - x$ en lugar de $\ln(x!)\approx x \ln x$ es que la aproximación de la fórmula más simple es tan mala que, a pesar de que funciona para $x\to\infty$ no funciona en el contexto físico con el que estamos jugando. Pero si esta es la razón, la elección debería ir acompañada de algunas consideraciones. Un vistazo a este gráfico sugiere que esta línea de pensamiento es correcta: incluso para un número enorme como el de Avogadro, la diferencia con las dos fórmulas es de aproximadamente 2\% (la relación se hizo mayor de 0,99 sólo para $2.688117141816367\cdot10^{43}$ )
Pero estas consideraciones no son suficientes para justificar el uso $\ln(x!)\approx x \ln x - x$ . Veo que $\ln(x!)\approx x \ln x$ no es apropiado para la prueba de la mecánica cuántica estadística, pero cómo puedo ver que $\ln(x!)\approx x \ln x - x$ ¿es apropiado? La única manera parece ser una comparación con la función $\ln(x!)$ pero aquí surgen problemas.
- Primero . Seguramente nos interesa un número grande y mucho más pequeño que el número de Avogadro, pero esto suena vago... ¿qué número debemos elegir como representativo de la población típica de un nivel de energía típico, para probar el buen funcionamiento de la fórmula aproximada?
- Segundo . Cuando el argumento de $\ln(x!)$ empieza a ser grande, pone a prueba incluso las potentes capacidades informáticas de los ordenadores
Hice esta trama
que sugiere que esta línea de pensamiento también podría ser correcta (para $x=170$ el valor máximo que mantiene la chatarra de mi ordenador antes de estrellarse, el error es de $0,5\%$ ), pero la trama no es decisiva y me gustaría algo más fuerte, que resolviera satisfactoriamente los dos problemas que enumeré arriba. Una forma de proceder podría ser encontrar una manera de maximizar el error cometido utilizando $x\ln x - x$ en lugar de $\ln(x!)$ y luego demostrar que este error no es significativo para cualquier valor razonable de $x$ pero no puedo hacerlo.
Adjunto
Me piden que escriba la prueba, informaré de la final: es posible mostrar bajo una suposición razonable que la estadística cuántica se puede encontrar encontrando el máximo (con restricciones de conservación de partículas y energía) de $Q$ , donde $Q$ para fermiones y bosones es respectivamente $$ Q_{ferm} = \prod_{n} \frac{d_n!}{N_n!(d_n-N_n)!} $$ $$ Q_{bos} = \prod_{n} \frac{(N_n+d_n-1)!}{N_n!(d_n-1)!} $$ y explotando la hipótesis $1 \ll N_n \ll d_n$ . Tomar el logaritmo no altera las posiciones de los máximos (este paso transforma los productos en sumas y nos permitirá explotar el Stirling). Aprovechando el método de Lagrange tenemos $$ \frac{\partial}{\partial N_n} \left[ \ln Q + \alpha \left( N - \sum_n N_n \right) + \beta \left( E_n - \sum_n N_n E_n \right) \right]=0 $$ Continuando llegamos al paso crucial, relacionado con esta cuestión: para los fermiones y bosones tenemos $$ \frac{\partial }{\partial N_n} \sum_n [ \ln ( d_n! ) - \ln ( N_n ! ) - \ln(d_N - N_n)! ] = \alpha + \beta E_n $$ $$ \frac{\partial }{\partial N_n} \sum_n [ \ln (N_n + d_n - 1)! - \ln ( N_n ! ) - \ln (d_n - 1)! ] = \alpha + \beta E_n $$ de la que se desprende que utilizando indistintamente $\ln(x!)\approx x \ln x$ o $\ln(x!)\approx x \ln x - x$ encontrará $N_n = \frac{d_n}{e^{\alpha+\beta e_n} \pm 1}$ . De todas formas este no es el verdadero problema que me preocupa, el problema es "cómo puedo ver que tomando dos términos en $\ln(x!) \approx \dots$ es razonablemente seguro?".
0 votos
¿Puede usted discutir que derivaciones en statmech a las que te refieres, y da ejemplos en los que la otra aproximación también funciona? Esto es probablemente más valioso que los gráficos que comparan la aproximación.
0 votos
Los comentarios no son para ampliar la discusión; esta conversación ha sido trasladado al chat .
0 votos
Me ha resultado más fácil entender algunas derivaciones que no utilizan la entropía y los logaritmos. Por ejemplo, es más fácil demostrar que el cambio fraccionario de la multiplicidad con la energía es una medida de la temperatura, y luego proceder que esto es lo mismo que ${\rm d} \ln \Omega/{\rm d}E$ por la regla de la cadena.
0 votos
@Pieter ¿dónde puedo encontrar tu prueba?
0 votos
@FaustoVezzaro No es una prueba rígida, pero escribí aquí: physics.stackexchange.com/questions/404797/ Y entonces se puede proceder a obtener el factor de Boltzmann: physics.stackexchange.com/questions/412215/